Minimum value of $sqrt {(x+2)^2+(y+2)^2}+sqrt {(x+1)^2+(y-1)^2}+sqrt{ (x-1)^2+(y+1)^2}$












2












$begingroup$


Let $x;yin R$. Find Minimum value of the function $$sqrt {(x+2)^2+(y+2)^2}+sqrt {(x+1)^2+(y-1)^2}+sqrt{ (x-1)^2+(y+1)^2}$$





My try: By Minkowski inequality:



$LHS=sqrt{(x+1)^2+(y-1)^2}+sqrt{(x-1)^2+(y+1)^2}+2sqrt{dfrac{1}{4}[(x+2)^2+(y+2)^2]}$



$=sqrt{(x+1)^2+(y-1)^2}+sqrt{(x-1)^2+(y+1)^2}+2sqrt{(dfrac{x}{2}+1)^2+(dfrac{y} {2}+1)^2]}$



$ge sqrt{(dfrac{3x}{2}+2)^2+(dfrac{3y}{2})^2}+sqrt {(dfrac{3y}{2}+2)^2+(dfrac{3x}{2})^2}ge sqrt{8}$



And the equality occurs when $x=-y-frac {4}{3}$



Help me check it. I fear that is not minimum value of the function. THx.










share|cite|improve this question









$endgroup$












  • $begingroup$
    math.stackexchange.com/questions/613028/…
    $endgroup$
    – lab bhattacharjee
    Jan 2 at 18:22










  • $begingroup$
    In my opinion it's not duplicate. The trying to find the solution of Nguyễn Duy Linh is much more better than all linked solutions.
    $endgroup$
    – Michael Rozenberg
    Jan 2 at 18:32












  • $begingroup$
    This function is symmetric. i.e. if you swap the x's and the y's you have the same function. That suggests that line $x=y$ is special
    $endgroup$
    – Doug M
    Jan 2 at 18:44






  • 2




    $begingroup$
    @MichaelRozenberg The function is symmetric. But that doesn't guarantee that that minimum lies on the line $x= y$ It could be that there is a minimum on one side and its pair on the other, with a saddle on the line $x=y$ Nonetheless, it gives a good place to begin the investigation.
    $endgroup$
    – Doug M
    Jan 2 at 18:55








  • 1




    $begingroup$
    Thx everyone, now i can solve it by Cauchy-Schwarz when i knew equality occurs when $x=y=frac{-1}{sqrt 3}$.
    $endgroup$
    – Nguyễn Duy Linh
    Jan 3 at 5:17
















2












$begingroup$


Let $x;yin R$. Find Minimum value of the function $$sqrt {(x+2)^2+(y+2)^2}+sqrt {(x+1)^2+(y-1)^2}+sqrt{ (x-1)^2+(y+1)^2}$$





My try: By Minkowski inequality:



$LHS=sqrt{(x+1)^2+(y-1)^2}+sqrt{(x-1)^2+(y+1)^2}+2sqrt{dfrac{1}{4}[(x+2)^2+(y+2)^2]}$



$=sqrt{(x+1)^2+(y-1)^2}+sqrt{(x-1)^2+(y+1)^2}+2sqrt{(dfrac{x}{2}+1)^2+(dfrac{y} {2}+1)^2]}$



$ge sqrt{(dfrac{3x}{2}+2)^2+(dfrac{3y}{2})^2}+sqrt {(dfrac{3y}{2}+2)^2+(dfrac{3x}{2})^2}ge sqrt{8}$



And the equality occurs when $x=-y-frac {4}{3}$



Help me check it. I fear that is not minimum value of the function. THx.










share|cite|improve this question









$endgroup$












  • $begingroup$
    math.stackexchange.com/questions/613028/…
    $endgroup$
    – lab bhattacharjee
    Jan 2 at 18:22










  • $begingroup$
    In my opinion it's not duplicate. The trying to find the solution of Nguyễn Duy Linh is much more better than all linked solutions.
    $endgroup$
    – Michael Rozenberg
    Jan 2 at 18:32












  • $begingroup$
    This function is symmetric. i.e. if you swap the x's and the y's you have the same function. That suggests that line $x=y$ is special
    $endgroup$
    – Doug M
    Jan 2 at 18:44






  • 2




    $begingroup$
    @MichaelRozenberg The function is symmetric. But that doesn't guarantee that that minimum lies on the line $x= y$ It could be that there is a minimum on one side and its pair on the other, with a saddle on the line $x=y$ Nonetheless, it gives a good place to begin the investigation.
    $endgroup$
    – Doug M
    Jan 2 at 18:55








  • 1




    $begingroup$
    Thx everyone, now i can solve it by Cauchy-Schwarz when i knew equality occurs when $x=y=frac{-1}{sqrt 3}$.
    $endgroup$
    – Nguyễn Duy Linh
    Jan 3 at 5:17














2












2








2


0



$begingroup$


Let $x;yin R$. Find Minimum value of the function $$sqrt {(x+2)^2+(y+2)^2}+sqrt {(x+1)^2+(y-1)^2}+sqrt{ (x-1)^2+(y+1)^2}$$





My try: By Minkowski inequality:



$LHS=sqrt{(x+1)^2+(y-1)^2}+sqrt{(x-1)^2+(y+1)^2}+2sqrt{dfrac{1}{4}[(x+2)^2+(y+2)^2]}$



$=sqrt{(x+1)^2+(y-1)^2}+sqrt{(x-1)^2+(y+1)^2}+2sqrt{(dfrac{x}{2}+1)^2+(dfrac{y} {2}+1)^2]}$



$ge sqrt{(dfrac{3x}{2}+2)^2+(dfrac{3y}{2})^2}+sqrt {(dfrac{3y}{2}+2)^2+(dfrac{3x}{2})^2}ge sqrt{8}$



And the equality occurs when $x=-y-frac {4}{3}$



Help me check it. I fear that is not minimum value of the function. THx.










share|cite|improve this question









$endgroup$




Let $x;yin R$. Find Minimum value of the function $$sqrt {(x+2)^2+(y+2)^2}+sqrt {(x+1)^2+(y-1)^2}+sqrt{ (x-1)^2+(y+1)^2}$$





My try: By Minkowski inequality:



$LHS=sqrt{(x+1)^2+(y-1)^2}+sqrt{(x-1)^2+(y+1)^2}+2sqrt{dfrac{1}{4}[(x+2)^2+(y+2)^2]}$



$=sqrt{(x+1)^2+(y-1)^2}+sqrt{(x-1)^2+(y+1)^2}+2sqrt{(dfrac{x}{2}+1)^2+(dfrac{y} {2}+1)^2]}$



$ge sqrt{(dfrac{3x}{2}+2)^2+(dfrac{3y}{2})^2}+sqrt {(dfrac{3y}{2}+2)^2+(dfrac{3x}{2})^2}ge sqrt{8}$



And the equality occurs when $x=-y-frac {4}{3}$



Help me check it. I fear that is not minimum value of the function. THx.







inequality radicals maxima-minima






share|cite|improve this question













share|cite|improve this question











share|cite|improve this question




share|cite|improve this question










asked Jan 2 at 18:16









Nguyễn Duy LinhNguyễn Duy Linh

1818




1818












  • $begingroup$
    math.stackexchange.com/questions/613028/…
    $endgroup$
    – lab bhattacharjee
    Jan 2 at 18:22










  • $begingroup$
    In my opinion it's not duplicate. The trying to find the solution of Nguyễn Duy Linh is much more better than all linked solutions.
    $endgroup$
    – Michael Rozenberg
    Jan 2 at 18:32












  • $begingroup$
    This function is symmetric. i.e. if you swap the x's and the y's you have the same function. That suggests that line $x=y$ is special
    $endgroup$
    – Doug M
    Jan 2 at 18:44






  • 2




    $begingroup$
    @MichaelRozenberg The function is symmetric. But that doesn't guarantee that that minimum lies on the line $x= y$ It could be that there is a minimum on one side and its pair on the other, with a saddle on the line $x=y$ Nonetheless, it gives a good place to begin the investigation.
    $endgroup$
    – Doug M
    Jan 2 at 18:55








  • 1




    $begingroup$
    Thx everyone, now i can solve it by Cauchy-Schwarz when i knew equality occurs when $x=y=frac{-1}{sqrt 3}$.
    $endgroup$
    – Nguyễn Duy Linh
    Jan 3 at 5:17


















  • $begingroup$
    math.stackexchange.com/questions/613028/…
    $endgroup$
    – lab bhattacharjee
    Jan 2 at 18:22










  • $begingroup$
    In my opinion it's not duplicate. The trying to find the solution of Nguyễn Duy Linh is much more better than all linked solutions.
    $endgroup$
    – Michael Rozenberg
    Jan 2 at 18:32












  • $begingroup$
    This function is symmetric. i.e. if you swap the x's and the y's you have the same function. That suggests that line $x=y$ is special
    $endgroup$
    – Doug M
    Jan 2 at 18:44






  • 2




    $begingroup$
    @MichaelRozenberg The function is symmetric. But that doesn't guarantee that that minimum lies on the line $x= y$ It could be that there is a minimum on one side and its pair on the other, with a saddle on the line $x=y$ Nonetheless, it gives a good place to begin the investigation.
    $endgroup$
    – Doug M
    Jan 2 at 18:55








  • 1




    $begingroup$
    Thx everyone, now i can solve it by Cauchy-Schwarz when i knew equality occurs when $x=y=frac{-1}{sqrt 3}$.
    $endgroup$
    – Nguyễn Duy Linh
    Jan 3 at 5:17
















$begingroup$
math.stackexchange.com/questions/613028/…
$endgroup$
– lab bhattacharjee
Jan 2 at 18:22




$begingroup$
math.stackexchange.com/questions/613028/…
$endgroup$
– lab bhattacharjee
Jan 2 at 18:22












$begingroup$
In my opinion it's not duplicate. The trying to find the solution of Nguyễn Duy Linh is much more better than all linked solutions.
$endgroup$
– Michael Rozenberg
Jan 2 at 18:32






$begingroup$
In my opinion it's not duplicate. The trying to find the solution of Nguyễn Duy Linh is much more better than all linked solutions.
$endgroup$
– Michael Rozenberg
Jan 2 at 18:32














$begingroup$
This function is symmetric. i.e. if you swap the x's and the y's you have the same function. That suggests that line $x=y$ is special
$endgroup$
– Doug M
Jan 2 at 18:44




$begingroup$
This function is symmetric. i.e. if you swap the x's and the y's you have the same function. That suggests that line $x=y$ is special
$endgroup$
– Doug M
Jan 2 at 18:44




2




2




$begingroup$
@MichaelRozenberg The function is symmetric. But that doesn't guarantee that that minimum lies on the line $x= y$ It could be that there is a minimum on one side and its pair on the other, with a saddle on the line $x=y$ Nonetheless, it gives a good place to begin the investigation.
$endgroup$
– Doug M
Jan 2 at 18:55






$begingroup$
@MichaelRozenberg The function is symmetric. But that doesn't guarantee that that minimum lies on the line $x= y$ It could be that there is a minimum on one side and its pair on the other, with a saddle on the line $x=y$ Nonetheless, it gives a good place to begin the investigation.
$endgroup$
– Doug M
Jan 2 at 18:55






1




1




$begingroup$
Thx everyone, now i can solve it by Cauchy-Schwarz when i knew equality occurs when $x=y=frac{-1}{sqrt 3}$.
$endgroup$
– Nguyễn Duy Linh
Jan 3 at 5:17




$begingroup$
Thx everyone, now i can solve it by Cauchy-Schwarz when i knew equality occurs when $x=y=frac{-1}{sqrt 3}$.
$endgroup$
– Nguyễn Duy Linh
Jan 3 at 5:17










1 Answer
1






active

oldest

votes


















1












$begingroup$

FERMAT POINT



I get, from the $120^circ$ construction, that the minimizing point is at $$ left(frac{-1}{sqrt 3},frac{-1}{sqrt 3} right)$$
The main calculation I did was $45^circ + 60^circ = 105^circ$ and $tan 105^circ = -2-sqrt 3.$ The sum of three distances is
$$ sqrt 6 + sqrt 8 approx 5.277916867529368195800661523 $$



enter image description here






share|cite|improve this answer











$endgroup$













  • $begingroup$
    It was closed as a duplicate of math.stackexchange.com/q/2046490/42969 – I cannot tell if those answers are right or wrong, but the result seems to be the same.
    $endgroup$
    – Martin R
    Jan 2 at 21:14










  • $begingroup$
    @MartinR Thank you. One of the answers there was correct, and was accepted.
    $endgroup$
    – Will Jagy
    Jan 2 at 21:18













Your Answer





StackExchange.ifUsing("editor", function () {
return StackExchange.using("mathjaxEditing", function () {
StackExchange.MarkdownEditor.creationCallbacks.add(function (editor, postfix) {
StackExchange.mathjaxEditing.prepareWmdForMathJax(editor, postfix, [["$", "$"], ["\\(","\\)"]]);
});
});
}, "mathjax-editing");

StackExchange.ready(function() {
var channelOptions = {
tags: "".split(" "),
id: "69"
};
initTagRenderer("".split(" "), "".split(" "), channelOptions);

StackExchange.using("externalEditor", function() {
// Have to fire editor after snippets, if snippets enabled
if (StackExchange.settings.snippets.snippetsEnabled) {
StackExchange.using("snippets", function() {
createEditor();
});
}
else {
createEditor();
}
});

function createEditor() {
StackExchange.prepareEditor({
heartbeatType: 'answer',
autoActivateHeartbeat: false,
convertImagesToLinks: true,
noModals: true,
showLowRepImageUploadWarning: true,
reputationToPostImages: 10,
bindNavPrevention: true,
postfix: "",
imageUploader: {
brandingHtml: "Powered by u003ca class="icon-imgur-white" href="https://imgur.com/"u003eu003c/au003e",
contentPolicyHtml: "User contributions licensed under u003ca href="https://creativecommons.org/licenses/by-sa/3.0/"u003ecc by-sa 3.0 with attribution requiredu003c/au003e u003ca href="https://stackoverflow.com/legal/content-policy"u003e(content policy)u003c/au003e",
allowUrls: true
},
noCode: true, onDemand: true,
discardSelector: ".discard-answer"
,immediatelyShowMarkdownHelp:true
});


}
});














draft saved

draft discarded


















StackExchange.ready(
function () {
StackExchange.openid.initPostLogin('.new-post-login', 'https%3a%2f%2fmath.stackexchange.com%2fquestions%2f3059781%2fminimum-value-of-sqrt-x22y22-sqrt-x12y-12-sqrt-x-12%23new-answer', 'question_page');
}
);

Post as a guest















Required, but never shown

























1 Answer
1






active

oldest

votes








1 Answer
1






active

oldest

votes









active

oldest

votes






active

oldest

votes









1












$begingroup$

FERMAT POINT



I get, from the $120^circ$ construction, that the minimizing point is at $$ left(frac{-1}{sqrt 3},frac{-1}{sqrt 3} right)$$
The main calculation I did was $45^circ + 60^circ = 105^circ$ and $tan 105^circ = -2-sqrt 3.$ The sum of three distances is
$$ sqrt 6 + sqrt 8 approx 5.277916867529368195800661523 $$



enter image description here






share|cite|improve this answer











$endgroup$













  • $begingroup$
    It was closed as a duplicate of math.stackexchange.com/q/2046490/42969 – I cannot tell if those answers are right or wrong, but the result seems to be the same.
    $endgroup$
    – Martin R
    Jan 2 at 21:14










  • $begingroup$
    @MartinR Thank you. One of the answers there was correct, and was accepted.
    $endgroup$
    – Will Jagy
    Jan 2 at 21:18


















1












$begingroup$

FERMAT POINT



I get, from the $120^circ$ construction, that the minimizing point is at $$ left(frac{-1}{sqrt 3},frac{-1}{sqrt 3} right)$$
The main calculation I did was $45^circ + 60^circ = 105^circ$ and $tan 105^circ = -2-sqrt 3.$ The sum of three distances is
$$ sqrt 6 + sqrt 8 approx 5.277916867529368195800661523 $$



enter image description here






share|cite|improve this answer











$endgroup$













  • $begingroup$
    It was closed as a duplicate of math.stackexchange.com/q/2046490/42969 – I cannot tell if those answers are right or wrong, but the result seems to be the same.
    $endgroup$
    – Martin R
    Jan 2 at 21:14










  • $begingroup$
    @MartinR Thank you. One of the answers there was correct, and was accepted.
    $endgroup$
    – Will Jagy
    Jan 2 at 21:18
















1












1








1





$begingroup$

FERMAT POINT



I get, from the $120^circ$ construction, that the minimizing point is at $$ left(frac{-1}{sqrt 3},frac{-1}{sqrt 3} right)$$
The main calculation I did was $45^circ + 60^circ = 105^circ$ and $tan 105^circ = -2-sqrt 3.$ The sum of three distances is
$$ sqrt 6 + sqrt 8 approx 5.277916867529368195800661523 $$



enter image description here






share|cite|improve this answer











$endgroup$



FERMAT POINT



I get, from the $120^circ$ construction, that the minimizing point is at $$ left(frac{-1}{sqrt 3},frac{-1}{sqrt 3} right)$$
The main calculation I did was $45^circ + 60^circ = 105^circ$ and $tan 105^circ = -2-sqrt 3.$ The sum of three distances is
$$ sqrt 6 + sqrt 8 approx 5.277916867529368195800661523 $$



enter image description here







share|cite|improve this answer














share|cite|improve this answer



share|cite|improve this answer








edited Jan 2 at 21:23

























answered Jan 2 at 19:43









Will JagyWill Jagy

103k5101200




103k5101200












  • $begingroup$
    It was closed as a duplicate of math.stackexchange.com/q/2046490/42969 – I cannot tell if those answers are right or wrong, but the result seems to be the same.
    $endgroup$
    – Martin R
    Jan 2 at 21:14










  • $begingroup$
    @MartinR Thank you. One of the answers there was correct, and was accepted.
    $endgroup$
    – Will Jagy
    Jan 2 at 21:18




















  • $begingroup$
    It was closed as a duplicate of math.stackexchange.com/q/2046490/42969 – I cannot tell if those answers are right or wrong, but the result seems to be the same.
    $endgroup$
    – Martin R
    Jan 2 at 21:14










  • $begingroup$
    @MartinR Thank you. One of the answers there was correct, and was accepted.
    $endgroup$
    – Will Jagy
    Jan 2 at 21:18


















$begingroup$
It was closed as a duplicate of math.stackexchange.com/q/2046490/42969 – I cannot tell if those answers are right or wrong, but the result seems to be the same.
$endgroup$
– Martin R
Jan 2 at 21:14




$begingroup$
It was closed as a duplicate of math.stackexchange.com/q/2046490/42969 – I cannot tell if those answers are right or wrong, but the result seems to be the same.
$endgroup$
– Martin R
Jan 2 at 21:14












$begingroup$
@MartinR Thank you. One of the answers there was correct, and was accepted.
$endgroup$
– Will Jagy
Jan 2 at 21:18






$begingroup$
@MartinR Thank you. One of the answers there was correct, and was accepted.
$endgroup$
– Will Jagy
Jan 2 at 21:18




















draft saved

draft discarded




















































Thanks for contributing an answer to Mathematics Stack Exchange!


  • Please be sure to answer the question. Provide details and share your research!

But avoid



  • Asking for help, clarification, or responding to other answers.

  • Making statements based on opinion; back them up with references or personal experience.


Use MathJax to format equations. MathJax reference.


To learn more, see our tips on writing great answers.




draft saved


draft discarded














StackExchange.ready(
function () {
StackExchange.openid.initPostLogin('.new-post-login', 'https%3a%2f%2fmath.stackexchange.com%2fquestions%2f3059781%2fminimum-value-of-sqrt-x22y22-sqrt-x12y-12-sqrt-x-12%23new-answer', 'question_page');
}
);

Post as a guest















Required, but never shown





















































Required, but never shown














Required, but never shown












Required, but never shown







Required, but never shown

































Required, but never shown














Required, but never shown












Required, but never shown







Required, but never shown







Popular posts from this blog

Human spaceflight

Can not write log (Is /dev/pts mounted?) - openpty in Ubuntu-on-Windows?

File:DeusFollowingSea.jpg